5
$\begingroup$

Fix $\epsilon>0$.

For all large $N$, does there exist $A\subset [N]:=\{1,\dots,N\}$ such that both $A+A$ and $A^c:=[N]\setminus A$ lack arithmetic progressions of length $N^\epsilon$?

I am aware Rusza used “niveau sets” to create dense subsets of $[N]$ whose sumsets lack progressions of length $\exp(\log^{2/3+o(1)}N)$ (see https://eudml.org/doc/206433). But I don’t understand the construction well enough to know if it lacks long progressions in the complement (and thus satisfies the above).

My motivation is that I read that it was an open problem to determine for $r\ge 2$ if there exists $\epsilon_r> 0$, so any $r$-coloring $c:[N]\to [r]$ has some color class $A$ where $A+A$ contains a progression of length $N^{\epsilon_r}$. If one could find some $\epsilon>0$ where my question is false, then we could inductively take $\epsilon_r =(r-1)\epsilon$. So I am hoping to learn if this approach is viable.

$\endgroup$

2 Answers 2

3
$\begingroup$

I don't know of a reference where this has been worked out for niveau sets in $[N]$, but I would guess that niveau sets do not work, by considering the 'finite field model' of niveau sets in $\mathbb{F}_2^n$ (as discussed by Ben Green in https://arxiv.org/pdf/math/0409420.pdf).

The 'niveau set' here is $A\subset \mathbb{F}_2^n$ consisting of all vectors with at least $n/2+\sqrt{n}/2$ ones. Green shows (Theorem 9.4 of the above paper) that $A+A$ does not contain any large cosets of subspaces, but the complement of $A$ (which is the set of all vectors with at least $n/2-\sqrt{n}/2$ zeros) trivially contains a subspace of dimension $n/2+\sqrt{n}/2$. (One can show by Green's argument that this is also best possible.)

This does not directly answer your question, but generally we expect $\mathbb{F}_2^n$ to behave similarly to $[N]$ and cosets of subspaces to be analogous to arithmetic progressions, and niveau sets behave similarly in both settings. So I would guess that the complements of Ruzsa's niveau sets in $[N]$ probably do contain arithmetic progressions of length $N^c$ for some $c>0$.

$\endgroup$
2
  • $\begingroup$ ah, thanks! that makes sense. $\endgroup$ Oct 4, 2022 at 10:42
  • 1
    $\begingroup$ an interesting thing to note is that the finite field model variant of my question has an easy construction. namely, by taking $A$ to be $2^{n/3}$ random elements, $A+A$ has codimension at least $n/3$ due to its cardinality, whilst $A^c$ should have codimension $(1/3-o(1))n$ with high probability. $\endgroup$ Oct 4, 2022 at 12:42
2
$\begingroup$

Ah, I realized how to construct such $A$.

The idea is to just crudely mimic the construction for the off-diagonal van der Waerden numbers $w(3,k)$.

We fix parameters $D,M,\rho$ which will be optimized later.

We shall sample $M$ points $x_1,\dots,x_M$ from the $D$-dimensional torus $\Bbb{T}^D := \Bbb{R}^D/\Bbb{Z}^D$. We then take $A_0\subset \Bbb{T}^D$ to be $\{x_1,\dots,x_M\}+B_0$ where $B_0$ is the box $[0,\rho]^D+\Bbb{Z}^D$.

One then chooses $\theta \in \Bbb{T}^D$ uniformly at random and takes $A=\{n\in[N]: n\theta \in A_0\}$. I claim that we can let $D=D(N)\to \infty$ as $N\to \infty$ while ensuring $A+A$ and $A^c$ both lack progressions of length $N^{O(1/D)}$.

In short, one just modifies several calculations from my paper (https://arxiv.org/abs/2111.01099). I might update with further details later, but if I’m not mistaken, this blocks progressions of length $\exp(O(\sqrt{\log N \log\log N}))$.

The intuition is that this looks like a union of $M$ generalized arithmetic progressions (GAP's) of rank $D$, $G_1,...,G_M$ where each $G_i$ "looks generic" and thus doesn't have any arithmetic progressions much longer than $N^{1/D}$. The $M$ GAP'S are sufficiently randomly distributed in $[N]$ that they block progressions in the complement, as was shown in the aforementioned paper.

Meanwhile, $A+A$ looks like a union of $M^2$ GAP's of rank $D$, which again all look generic. Taking $M$ much smaller than $N^{1/D}$, these $M^2$ GAP's in $A+A$ shouldn't really interact, resulting in there not being progressions dramatically longer than $N^{1/D}$.

$\endgroup$

Your Answer

By clicking “Post Your Answer”, you agree to our terms of service and acknowledge that you have read and understand our privacy policy and code of conduct.

Not the answer you're looking for? Browse other questions tagged or ask your own question.